A track has the dimensions shown.
36.5 m
ISTAN
SAMANT
men komm
84.4 m
Ta
inside of track
outside of track
. The track has 8 lanes
• Each lane is 2.1 meters wide
36.5 m
O
TI
16. To the nearest tenth of a meter, what is
the perimeter of the outside of the
track?
Byp
*REQUIRED
ANA
1
√x
Sign out

A Track Has The Dimensions Shown.36.5 MISTANSAMANTmen Komm84.4 MTainside Of Trackoutside Of Track. The

Answers

Answer 1

Answer:

Step-by-step explanation:


Related Questions

Now that you have chosen your mode of transportation, use your choice to answer the questions that follow.





What would the cost of your transportation be if you drove:

a. 10 miles? b. 25 miles? c. 42 miles? d. 68 miles?

Make sure to list your chosen mode of transportation and then answer all parts and show your work

Answers

(a) The cost of City Bus for driving 10 miles = $3.

(b) The cost of City Bus for driving 25 miles = $7.5.

(c) The cost of City Bus for driving 42 miles = $12.6.

(d) The cost of City Bus for driving 68 miles = $20.4.

We previously choose City Bus as our mode transport since the per mile cost for City Bus is less.

Let the model for City Bus be f(x) = cx + d, where f(x) is total cost and x is number of miles.

From the table of Taxi we get, f(2) = 0.60; f(4) = 1.20; f(6) = 1.80 and f(8) = 2.40.

So, 2a + b = 0.60 and 4a + b = 1.20

(4a + b) - (2a + b) = 1.20 - 0.60

2a = 0.60

a = 0.60/2 = 0.30

Now, f(8) = 2.40

8*0.30 + b = 2.40

2.40 + b = 2.40

b = 2.40 - 2.40 = 0

So the function rule for City Bus is, f(x) = 0.3x.

(a) Total cost to drive 10 miles is,

f(10) = 0.3*10 = 3

(b) Total cost to drive 25 miles is,

f(25) = 0.3*25 = 7.5

(c) Total cost to drive 42 miles is,

f(42) = 0.3*42 = 12.6

(d) Total cost to drive 68 miles is,

f(68) = 0.3*68 = 20.4

To know more about function rule here

https://brainly.com/question/30139621

#SPJ1

In a regular tiling, if there are six polygons meeting at a vertex, then the angles at the vertex are _____ degrees

Answers

In a regular tiling, if there are six polygons meeting at a vertex, then the angles at the vertex are 120 degrees.

This is because each regular polygon has interior angles that are multiples of 180 degrees divided by the number of sides. For a regular hexagon, which has six sides, each interior angle measures 120 degrees. When six regular hexagons meet at a vertex in a regular tiling, the total angle sum at the vertex is 720 degrees (6 times 120 degrees).

Since the angles must be divided equally among the six hexagons, each angle at the vertex is 120 degrees.

Learn more about interior angles:

https://brainly.com/question/24966296

#SPJ11

find the extremes of 4x−4y subject to condition x2 + 2y2 = 1

Answers

To find the extremes of 4x−4y subject to the condition x2 + 2y2 = 1, we can use the method of Lagrange multipliers.

First, we set up the Lagrange equation:

∇f(x,y) = λ∇g(x,y)

where f(x,y) = 4x-4y and g(x,y) = x2 + 2y2 - 1.

Taking partial derivatives, we have:

∂f/∂x = 4
∂f/∂y = -4
∂g/∂x = 2x
∂g/∂y = 4y

Setting these equal to their respective Lagrange multipliers, we have:

4 = 2λx
-4 = 4λy
x2 + 2y2 = 1

Solving for x and y in terms of λ, we get:

x = 2λ/4 = λ/2
y = -λ/4

Substituting these back into the constraint equation, we have:

(λ/2)2 + 2(-λ/4)2 = 1
λ2/4 + λ2/8 = 1
3λ2/8 = 1
λ2 = 8/3

Taking the positive and negative square roots of λ2, we have:

λ = ±2√2/3

Substituting these values back into x and y, we get:

For λ = 2√2/3:
x = (2√2/3)/2 = √2/3
y = -(2√2/3)/4 = -√2/6

For λ = -2√2/3:
x = (-2√2/3)/2 = -√2/3
y = -(-2√2/3)/4 = √2/6

Now we can find the extreme values of f(x,y) by plugging in these values of x and y:

f(√2/3, -√2/6) = 4(√2/3) - 4(-√2/6) = 4√2
f(-√2/3, √2/6) = 4(-√2/3) - 4(√2/6) = -4√2

Therefore, the maximum value of 4x-4y subject to the condition x2 + 2y2 = 1 is 4√2 and the minimum value is -4√2.

To learn more about derivative  visit;

brainly.com/question/30365299

#SPJ11

Show that the two straight lines through the origin which make an angle 45° with the line px + qy + r = 0 are given by the equation (p²-q)(x² - y²) + 4pqxy = 0. ​

Answers

The equation of the two straight lines through the origin making an angle of 45° with the line px + qy + r = 0 is (p²-q)(x² - y²) + 4pqxy = 0.

How to show the equation for the two straight lines passing through the origin and making a 45° angle with the line px + qy + r = 0?

To prove that the equation of the two straight lines through the origin, making an angle of 45° with the line px + qy + r = 0, is given by (p²-q)(x² - y²) + 4pqxy = 0, we can use the concept of slopes and trigonometric identities.

Let's consider the line px + qy + r = 0. The slope of this line is given by -p/q.

Now, the lines making an angle of 45° with this line will have slopes equal to tan(45°), which is 1.

Using the formula for the tangent of the sum of angles, we have:

tan(45°) = (m - (-p/q))/(1 + m(-p/q)), where m represents the slope of one of the lines.

Simplifying the equation, we get:

1 = (mq + p)/(q - mp)

Cross-multiplying and rearranging the terms, we obtain:

(p² - q)(m² - 1) + 2pqm = 0

Since these lines pass through the origin (0,0), we can replace m with y/x. Substituting y/x for m in the equation above, we get:

(p² - q)(x² - y²) + 2pqxy = 0

Further simplifying the equation, we arrive at:

(p² - q)(x² - y²) + 4pqxy = 0

Hence, we have proven that the equation of the two straight lines through the origin, making an angle of 45° with the line px + qy + r = 0, is given by (p²-q)(x² - y²) + 4pqxy = 0.

Learn more about origin

brainly.com/question/28490920

#SPJ11

A portion of an electrical circuit is displayed next. the switches operate independently of each other, and the probability that each relay closes when the switch is thrown is displayed by the switch. what is the probability that current will flow from s to t when the switch is thrown

Answers

If you provide me with a specific circuit diagram and the relevant details, I would be happy to help you determine the probability of current flowing from s to t when the switch is thrown.

What is the probability of current flowing from s to t when the switch is thrown?

I apologize, but it seems that the circuit diagram you mentioned is not displayed here. Without the circuit diagram, it is not possible for me to provide a specific answer to your question.

However, in general, the probability of current flowing from s to t in an electrical circuit depends on several factors such as the voltage level, the resistance of the circuit components, and the state of the switches. If the switches are all closed, then the probability of current flowing from s to t will depend on the overall resistance of the circuit.

If you provide me with a specific circuit diagram and the relevant details, I would be happy to help you determine the probability of current flowing from s to t when the switch is thrown.

Learn more about specific circuit

brainly.com/question/30888688

#SPJ11

Find the line of tangency to the circle defined by (x-3)^2 + (y-7)^2 = 169 at the point (15,2).

Answers

first off, let's look at the equation of the circle

[tex]\textit{equation of a circle}\\\\ (x- h)^2+(y- k)^2= r^2 \hspace{5em}\stackrel{center}{(\underset{}{h}~~,~~\underset{}{k})}\qquad \stackrel{radius}{\underset{}{r}} \\\\[-0.35em] ~\dotfill\\\\ (x-\stackrel{h}{3})^2+(y-\stackrel{k}{7})=169\implies (x-\stackrel{h}{3})^2+(y-\stackrel{k}{7})=\stackrel{ r }{13^2}[/tex]

so we have a circle centered at (3 , 7) with a radius of 13, Check the picture below.

so the line we want is the line in purple, which is tangential to the circle and therefore perpendicular to the blue line.

keeping in mind that perpendicular lines have negative reciprocal slopes, let's check for the slope of the blue line

[tex](\stackrel{x_1}{3}~,~\stackrel{y_1}{7})\qquad (\stackrel{x_2}{15}~,~\stackrel{y_2}{2}) ~\hfill \stackrel{slope}{m}\implies \cfrac{\stackrel{\textit{\large rise}} {\stackrel{y_2}{2}-\stackrel{y1}{7}}}{\underset{\textit{\large run}} {\underset{x_2}{15}-\underset{x_1}{3}}} \implies \cfrac{ -5 }{ 12 } \implies - \cfrac{5 }{ 12 } \\\\[-0.35em] ~\dotfill[/tex]

[tex]\stackrel{~\hspace{5em}\textit{perpendicular lines have \underline{negative reciprocal} slopes}~\hspace{5em}} {\stackrel{slope}{ \cfrac{-5}{12}} ~\hfill \stackrel{reciprocal}{\cfrac{12}{-5}} ~\hfill \stackrel{negative~reciprocal}{-\cfrac{12}{-5} \implies \cfrac{12}{ 5 }}}[/tex]

so we're really looking for the equation of a line whose slope is 12/5 and it passes through (15 , 2)

[tex](\stackrel{x_1}{15}~,~\stackrel{y_1}{2})\hspace{10em} \stackrel{slope}{m} ~=~ \cfrac{12}{5} \\\\\\ \begin{array}{|c|ll} \cline{1-1} \textit{point-slope form}\\ \cline{1-1} \\ y-y_1=m(x-x_1) \\\\ \cline{1-1} \end{array}\implies y-\stackrel{y_1}{2}=\stackrel{m}{ \cfrac{12}{5}}(x-\stackrel{x_1}{15}) \\\\\\ y-2=\cfrac{12}{5}x-36\implies {\Large \begin{array}{llll} y=\cfrac{12}{5}x-34 \end{array}}[/tex]

Quadrilateral FGHJ was dilated with the origin as the center of dilation to create quadrilateral F' G′ H′ J′.



Which rule best represents the dilation that was applied to quadrilateral FGHJ to create quadrilateral F' G′ H′ J′?



A. (x, y) à (5/7x, 5/7y)


B. (x, y) à (1. 4x , 1. 4y)


C. (x, y) à (x + 1, y + 2)


D. (x, y) à (x - 2, y + 1)









Which rule best represents the dilation that was applied to quadrilateral FGHJ to create quadrilateral F' G′ H′ J′?

Answers

The rule that best represents the dilation that was applied to quadrilateral FGHJ to create quadrilateral F'G'H'J' is option B, which is (x, y) à (1.4x, 1.4y).

What is the dilation rule used to create quadrilateral F'G'H'J' from FGHJ?

A dilation is a transformation that changes the size of an object without changing its shape. It is performed by multiplying the coordinates of each point by a scale factor.

In this case, the center of dilation is the origin, which means that the coordinates of each point are multiplied by the same scale factor in both the x and y directions.

The scale factor can be found by comparing the corresponding side lengths of the two quadrilaterals. In this case, the scale factor is 1.4, which means that the lengths of the sides of F'G'H'J' are 1.4 times the lengths of the corresponding sides of FGHJ.

Learn more about dilation

brainly.com/question/13176891

#SPJ11

In the last 215 days, builders have completed 700 m2 of the alligator habitat that will eventually be 1,200 m2. How much longer will it take to complete the alligator habitat?

Answers

In the last 215 days, builders have completed 700 m2 of the alligator habitat that will eventually be 1,200 m2.
It will take approximately 153 days to complete the remaining part of the alligator habitat.

Determine how much longer it will take to complete the alligator habitat, first, we need to find the rate at which the builders are working.
Calculate the work rate
The builders have completed 700 m2 of the 1,200 m2 alligator habitat in 215 days.
Work rate = (completed work) / (number of days)
Work rate = 700 m2 / 215 days = 3.26 m2/day (approximately)
Calculate the remaining work
The total area of the alligator habitat is 1,200 m2, and 700 m2 has been completed.
Remaining work = Total area - Completed work
Remaining work = 1,200 m2 - 700 m2 = 500 m2
Calculate the time to complete the remaining work
Time to complete = (remaining work) / (work rate)
Time to complete = 500 m2 / 3.26 m2/day ≈ 153.37 days
It will take approximately 153 days to complete the remaining part of the alligator habitat.

Read more about alligator habitat.

https://brainly.com/question/15823089

#SPJ11

It will take approximately 394 more days to complete the alligator habitat.

We can start by finding the proportion of the habitat that has already been completed:

proportion completed = 700 m^2 / 1200 m^2 = 0.5833

This means that there is still 1 - 0.5833 = 0.4167 (or 41.67%) of the habitat left to complete.

Next, we can use a proportion to find out how long it will take to complete the remaining 41.67% of the habitat:

215 days / 0.5833 = x days / 0.4167

Solving for x, we get:

x = 215 days * 0.4167 / 0.5833 ≈ 153 days

Therefore, the total time it will take to complete the alligator habitat is approximately 215 + 153 = 368 days, or about 394 more days from the start.

For more questions like Alligator click the link below:

https://brainly.com/question/154098

#SPJ11

Roxie plans on purchasing a new desktop computer for $1250. Which loan description would result in the smallest monthly payment when she pays the loan back?



12 months at 6. 25% annual simple interest rate



18 months at 6. 75% annual simple interest rate



24 months at 6. 5% annual simple interest rate



30 months at 6. 00% annual simple interest rate

Answers

The loan with the smallest monthly payment is the 30-month loan at 6% annual simple interest rate, with a monthly payment of $45.83.

To determine the loan with the smallest monthly payment, we need to calculate the monthly payment for each loan option and compare them.

We can use the formula for monthly payment on a simple interest loan:

monthly payment = (principal + (principal * interest rate * time)) / total number of payments

where:

principal is the amount borrowed (in this case, $1250)interest rate is the annual simple interest rate divided by 12 to get the monthly ratetime is the length of the loan in months

We can compute the monthly payments for each loan choice using this formula:

1. 12 Monthly interest rate = 0.0625/12 = 0.00521, monthly payment = (1250 + (1250 * 0.00521 * 12)) / 12 = $107.35

2. 18 months at 6.75%: monthly interest rate = 0.0675/12 = 0.00563, monthly payment = (1250 + (1250 * 0.00563 * 18)) / 18 = $81.96

3. 24 months at 6.5%: monthly interest rate = 0.065/12 = 0.00542, monthly payment = (1250 + (1250 * 0.00542 * 24)) / 24 = $66.14

4. 30 months at 6%: monthly interest rate = 0.06/12 = 0.005, monthly payment = (1250 + (1250 * 0.005 * 30)) / 30 = $45.83

Based on these calculations, the loan with the smallest monthly payment is the 30-month loan at 6% annual simple interest rate, with a monthly payment of $45.83.

Read more on simple interest on:

brainly.com/question/24429956

#SPJ4

help please ill give brainliest

Answers

In the given circle, measure of angle m is 44° and the measure of angle n is 39°. Thus, the value of m is 44 and the value of n is 39

Circle Geometry: Calculating the values of m and n

From the question, we are to determine the values of m and n in the given circle

From one of the circle theorems, we have that

The angles at the circumference subtended by the same arc are equal. That is, angles in the same segment are equal.

In the given diagram,

Angle m is in the same segment as the angle that measures 44°

Since angles in the same segment are equal,

Measure of angle m = 44°

Also,

Angle n is in the same segment as the angle that measures 39°

Since angles in the same segment are equal,

Measure of angle n = 39°

Hence,

m ∠m = 44°

m ∠n = 39°

Learn more on Circle Geometry here: https://brainly.com/question/29911839

#SPJ1

1 pts How much bubble wrap is needed to cover a cylindrical vase that is 16 inches tall with a diameter of 6 inches?​

Answers

415 square inches of bubble wrap to cover the cylindrical vase that is 16 inches tall with a diameter of 6 inches.

To calculate how much bubble wrap is needed to cover the cylindrical vase, you will need to find the circumference and height of the vase.

First, calculate the circumference of the vase using the diameter of 6 inches:
Circumference = π x diameter
Circumference = 3.14 x 6
Circumference = 18.84 inches

Next, calculate the height of the vase which is given as 16 inches.

To find the surface area of the vase, you will need to multiply the circumference by the height and add the area of the circular bases. The formula for the surface area of a cylinder is:

Surface area = 2πr² + 2πrh
where r is the radius and h is the height.

Since the vase has circular bases, we can find the area of each base by using the formula:
Area of circle = πr²

Now, let's find the radius of the vase:
[tex]Radius = \frac{diameter}{2}[/tex]
[tex]Radius = \frac{6}{2}[/tex]
Radius = 3 inches

So, the area of each base is:

Area of base = π x (radius)²
Area of base = π x 3²
Area of base = 28.27 square inches

The total area of the two bases is 2 x 28.27 = 56.54 square inches.

Now, let's find the surface area of the cylinder:

Surface area = 2πr² + 2πrh
Surface area = 2 x π x 3² + 2 x π x 3 x 16
Surface area = 113.1 + 301.44
Surface area = 414.54 square inches

Therefore, you would need approximately 415 square inches of bubble wrap to cover the cylindrical vase that is 16 inches tall with a diameter of 6 inches.

To know more about "surface area of a cylinder" refer here:

https://brainly.com/question/28575608#

#SPJ11

12
Find the first and second derivatives. S = 15 + 344 - 1 15 S' = S'' =

Answers

The first derivative of S is S' = 1/15.
The second derivative of S is S'' = 0.



To find the first derivative (S'):

Starting with the given equation S = 15 + 344 - 1 15, we can simplify it to S = 344 + 15.

We can take the derivative of each term separately since they are added together.

The derivative of a constant (15 and 344) is always 0, so we only need to take the derivative of 1/15.

S' = d/dx (344 + 15)

= d/dx (359)

= 0 + 0 + (d/dx (1/15))

= 1/15

Therefore, the first derivative of S is S' = 1/15.

To find the second derivative (S''):

We need to take the derivative of the first derivative (S').

Since the derivative of a constant is always 0,

we only need to take the derivative of 1/15.

S'' = d/dx (S')

= d/dx (1/15)

= 0

Therefore, the second derivative of S is S'' = 0.

To know more about Derivatives:

https://brainly.com/question/25324584

#SPJ11

Casho went shopping for a new pair of sneakers because of a sale. The price on the tag was $25, but Casho paid $22. 50 before tax. Find the percent discount

Answers

The percent discount on the sneakers is 10%

Casho paid $22.50 before tax, despite the item's $25 tag price. The discount is the difference between the original price and the sale price, which is $25 - $22.50 = $2.50.

The discount is the difference between the original price and the discounted price, expressed as a percentage of the original price.

To find the percent discount, we divide the discount by the original price and multiply by 100:

Percent discount = (discount / tag price) x 100

Percent discount = ($2.50 / $25) x 100

Percent discount = 0.1 x 100

Percent discount = 10%

Therefore, the percent discount on the sneakers is 10%

Learn more about Discount here

https://brainly.com/question/3541148

#SPJ4

Please help with the 2nd one

Answers

Answer:

Step-by-step explanation:

1807

A survey was taken by students in 6th, 7th, and 8th grade to determine how many first cousins they have. The results are shown in the box plots below. Use these box plots to answer the questions.

Answers

8th grade has the highest amount of first cousins

7. A rectangular prism has a volume of 135ft^3. The width of the rectangular prism is (2x+10)ft. The height of the rectangular prism is 5 times it's width. Write a expression that gives the length of the rectangular prism in feet?



A. 4(x+5)/27 B. 27/4(x+5)



C. (2x^2+100)/27. D. 27/(2x^2+100)

Answers

The expression that gives the length of the rectangular prism in feet is option D: 27/(2x^2+100).

What is the expression that gives the length of the rectangular prism in feet?

The volume of a rectangular prism is given by the formula V = lwh, where l is the length, w is the width, and h is the height.

We are given that the volume of the rectangular prism is 135ft^3, and the width is (2x+10)ft. Also, the height is 5 times the width, so h = 5w.

Substituting these values in the formula for the volume, we get:

135 = l(2x+10)(5w)

Dividing both sides by (2x+10)(5w), we get:

l = 135 / (2x+10)(5w)

l = 135 / [10(x+5)w]

Now we can substitute h = 5w:

l = 135 / [10(x+5)h/5]

l = 135 / [2(x+5)h]

l = 135 / [2(x+5)(5w)]

l = 135 / [10(x+5)^2]

Simplifying the expression, we get:

l = 27 / (2(x+5)^2)

Therefore, the expression that gives the length of the rectangular prism in feet is option D: 27/(2x^2+100).

Learn more about rectangular prism in feet

brainly.com/question/30956442

#SPJ11

do you believe your children will have a higher standard of living than you have? this question was asked of a national sample of american adults with children in time/cnn poll. sixty-three percent answered in the affirmatve, with a margin of error or plys or minus 3%. assume that the true percentage of all american adults who beleive their children with have a hgiehr standard of living is .60

Answers

True percentage of all American believes that their children have higher standard of living with confidence interval of 95% is between 60% and 66% .

CI is the confidence interval

Answered in the affirmative =  63%

p is the sample proportion =0.63

z is the critical value from the standard normal distribution at the desired confidence level

Using attached z-score table,

95% confidence level corresponds to z=1.96

n is the sample size

Use the margin of error ,

Calculate a confidence interval for percentage of American adults who believe their children will have a higher standard of living.

A margin of error of plus or minus 3% means ,

95% confident that the true percentage falls within 3% of the sample percentage.

Using the formula for a confidence interval for a population proportion,

CI = p ± z×√(p(1-p)/n)

Plugging in the values, we get,

⇒ CI = 0.63 ± 1.96√(0.63(1-0.63)/n)

Solving for n, we get,

n = (1.96/0.03)^2 × 0.63(1-0.63)

⇒ n = 994.87

Rounding up to the nearest whole number, sample size of at least 995.

⇒ CI = 0.63 ± 1.96√(0.63(1-0.63)/995)

⇒CI = 0.63 ± 0.02999

95% confidence interval for the true percentage is,

⇒CI = 0.63 ± 0.03

⇒CI = (0.60, 0.66)

Therefore, 95% confidence interval that between 60% and 66% of all American adults with children believe that their children will have a higher standard of living.

Learn more about confidence interval here

brainly.com/question/16409874

#SPJ4

The width of the large size is 9.9 cm and its height is 19.8 cm.
The width of the small size bottle is 4.5 cm.
hcm
h =
4.5 cm
Calculate the height of the small bottle.
19.8 cm
9.9 cm
+
cm

Answers

Answer and Explanation:

The height of the small bottle can be calculated using the ratio of the width of the large and small bottles.

Ratio of width = Large bottle width / Small bottle width

Ratio of width = 9.9 cm / 4.5 cm

Ratio of width = 2.2

Therefore, the height of the small bottle can be calculated by multiplying the ratio of width with the height of the large bottle.

Height of small bottle = Ratio of width x Height of large bottle

Height of small bottle = 2.2 x 19.8 cm

Height of small bottle = 43.56 cm

Find the solution and also verify your answer , under root 12 x 12 x - 4 is equals under root 4 x + 8

Answers

The solution to the given equation is x = -3/4 or x = 1.

What values of x satisfy the equation √(12x² - 4) = √(4x + 8)?

In order to find the solution, we start by squaring both sides of the equation to eliminate the square roots:

12x² - 4 = 4x + 8

Next, we simplify the equation by moving all the terms to one side:

12x² - 4x - 12 = 0

Now we can factor the quadratic equation:

4x² - x - 3 = 0

By factoring or using the quadratic formula, we find that the equation can be written as:

(4x + 3)(x - 1) = 0

Setting each factor equal to zero gives us the solutions:

4x + 3 = 0 or x - 1 = 0

Solving for x in each equation yields:

x = -3/4 or x = 1

Therefore, the solution to the given equation is x = -3/4 or x = 1.

Learn more about square roots

brainly.com/question/29286039

#SPJ11

If one line passes through the points (-3,8) & (1,9), and a perpendicular line passes through the point (-2,4), what is another point that would lie on the 2nd line. Select all that apply. ​

Answers

One point that would lie on the second line is (0,-4). Another  possible point on the 2nd line is (0, 12).

To find the equation of the first line, we can use the slope-intercept form:

y = mx + b

where m is the slope and b is the y-intercept. The slope of the line passing through (-3,8) and (1,9) can be found using the formula:

m = (y2 - y1) / (x2 - x1)

m = (9 - 8) / (1 - (-3))

m = 1/4

Using one of the points and the slope, we can find the y-intercept:

8 = (1/4)(-3) + b

b = 9

So the equation of the first line is:

y = (1/4)x + 9

To find the equation of the second line, we need to use the fact that it is perpendicular to the first line. The slopes of perpendicular lines are negative reciprocals, so the slope of the second line is:

m2 = -1/m1 = -1/(1/4) = -4

Using the point-slope form, we can write the equation of the second line:

y - 4 = -4(x + 2)

y - 4 = -4x - 8

y = -4x - 4

To find a point that lies on this line, we can plug in a value for x and solve for y. For example, if we let x = 0, then:

y = -4(0) - 4

y = -4

So the point (0,-4) lies on the second line.

Therefore, another point that would lie on the second line is (0,-4).

More on points: https://brainly.com/question/29395724

#SPJ11

The length of a rectangle is 6 ft longer than its width. if the perimeter of the rectangle is 64 ft, find its length and width

Answers

The length of the rectangle is 19 feet and its width is 13 feet.

Let's denote the width of the rectangle by w. Then, according to the problem statement, the length of the rectangle is 6 feet longer, which means it is equal to w + 6.

The perimeter of a rectangle is given by the formula:

perimeter = 2 × length + 2 × width

Substituting the expressions for length and width that we have just found, we get:

64 = 2 × (w + 6) + 2w

Simplifying the right-hand side:

64 = 2w + 12 + 2w

64 = 4w + 12

52 = 4w

w = 13

So the width of the rectangle is 13 feet. Using the expression for the length we found earlier, the length is:

length = w + 6 = 13 + 6 = 19

Therefore, the length of the rectangle is 19 feet and its width is 13 feet.

To know more about rectangle, refer to the link below:

https://brainly.com/question/28711757#

#SPJ11

Suppose that in 1682, a man bought a diamond for $32. Suppose that the man had instead put the $32 in the bank at 3% interest compounded continuously. What would that $32 have been worth in 2003? In 2003, the $32 would have been worth $ (Do not round until the final answer. Then round to the nearest dollar as needed.)

Answers

If a man bought a diamond for $32 in 1682 and the man had instead put the $32 in the bank at 3% interest compounded continuously, then the value of the diamond in 2003 would be $554,311.

The given problem is related to exponential growth. In this problem, the continuous compounding formula will be used to find the value of $32 in 2003.

The formula for continuous compounding is given by:

A = Pert   Where,

P is the principal amount,

r is the annual interest rate,

e is the Euler's number which is approximately 2.71828, and

t is the time in years.

Using the formula, we get:

A = 32e^(0.03 x 321)

A = 32e^9.63

A = 32 x 17322.23

A = $ 554311.36

Thus, $32 invested at 3% compounded continuously from 1682 to 2003 would be worth $554,311.

To know more about interest, click on the link below:

https://brainly.com/question/30393144#

#SPJ11

(n+3)!/(n+1)! please help immediately

Answers

Answer:

(n + 3)(n + 2) or n² + 5n + 6

------------------

The factorial of a non-negative integer n, denoted by n!, is the product of all positive integers less than or equal to n:

n! = n × (n - 2) × (n - 3) × ... × 2 × 1

As per above mentioned definition we see that:

(n + 3)! = (n + 3) × (n + 2) × (n + 1)!

Hence the quotient of (n + 3)! and (n + 1)! is:

(n + 3)(n + 2) or n² + 5n + 6

Sergey is solving 5x2 + 20x – 7 = 0. Which steps could he use to solve the quadratic equation by completing the square? Select three options

Answers

The quadratic equation where the squares had been completed is:

(x + 2)² = 27/5

How to complete squares?

Remember the perfect square trinomial:

(a + b)² = a² + 2ab + b²

now we have the quadratic equation:

5x² + 20x - 7 = 0

If we divide it all by 5, we will get.

x² + 4x - 7/5 = 0

Now we can rewrite this as:

(x² + 2*2*x )  - 7/5 = 0

Now we need to add 2² in both sides, we will get:

(x² + 2*2x + 2²) - 7/5 = 2²

(x + 2)² = 4 + 7/5

(x + 2)² = 27/5

There the square is completed.

Learn more about completing squares at:

https://brainly.com/question/10449635

#SPJ1

A new sign is being designed for the cityâs skate park. Knowing the exact angles is necessary for fitting the sign where it will hang. The architect started to write in the angles, but went home sick before she could finish. It is up to you to fill in the missing angles. For 4 of the 8 missing angles, explain your answer

Answers

Using trigonometry, we can solve for the missing angles to find them as 18.43°, 45°, 45°, and 18.43°.

The sign is mounted on a sloped surface, which means that we'll need to use some trigonometry to find the missing angles.

Let's concentrate on the sign's upper right corner, where the letters x and y are absent from two perspectives. The magnitude of angle x may be determined using trigonometry.

Let's begin by sketching a right triangle that has an angle x. The triangle's two sides may be represented by the sign's vertical and horizontal lines, with the addition of a third side to join the top right corner of the sign to the sloping area below.

Since the sign is an octagon, we know that each interior angle is 135°. Therefore, the measure of angle y must be:

y = 180 - 135 = 45°

Now, let's look at the right triangle that includes angle x. We know that the hypotenuse of the triangle is the sloped surface of the sign, which has a length of 4.5 meters. We also know that the opposite side of the triangle is the height of the sign above the ground, which has a length of 1.5 meters.

Using trigonometry, we can find the measure of angle x by taking the inverse tangent of the opposite side over the adjacent side:

tan(x) = opposite/adjacent = 1.5/4.5 = 1/3

x = tan⁻¹(1/3) ≈ 18.43°

Therefore, the measure of angle x is approximately 18.43 degrees.

Hence, using trigonometry, we can solve for the missing angles to find them as 18.43°, 45°, 45°, and 18.43°.

To learn more about trigonometry, refer to:

https://brainly.in/question/2685053

#SPJ4

Please help ASAP!!!!! In a certain Spanish class of 30 students, 11 of them play basketball and 15 of them play baseball. There are 10 students who play both sports. What is the probability that a student chosen randomly from the class plays basketball or baseball? Answer


should be a fraction in simplest form

Answers

The probability that a student chosen randomly from the class plays basketball or baseball is 8/15

Total number of students in Spanish class = 30

Student who plays basketball (A) = 11

Student who plays baseball (B) = 15

Student who plays both sports (A and B) = 10

To find a student who plays basketball or baseball (A or B)

(A or B)  = A + B -  (A and B)

(A or B) = 11 +15 -10

(A or B) = 16

P(A or B) = No. of favorable outcome/ Total no. of outcomes

P(A or B) = 16/30

In simplest form = 8/15

To know more about probability click here :

https://brainly.com/question/30034780

#SPJ4

After a windstorm, a leaning pole makes a 75° angle with the road surface. the pole casts a 15-foot shadow when the sun is at a 45° angle of elevation. about how long is the pole?

Answers

The pole is approximately 3.86 feet tall.

What is the length of a leaning pole that makes a 75° angle with the road surface, if it casts a 15-foot shadow when the sun is at a 45° angle of elevation?

Let's denote the height of the pole as "x" (in feet). From the problem, we know that the pole makes a 75° angle with the road surface, which means that the angle between the pole and the vertical is 90° - 75° = 15°.

Now, we can use the tangent function to find the height of the pole:

tan(15°) = x/15

Multiplying both sides by 15, we get:

x = 15 tan(15°) ≈ 3.86 feet

Learn more abut length

brainly.com/question/9842733

#SPJ11

900,000=x+y+z
79,750=0. 08x+0. 09y+0. 01z
2x=z

Answers

Answer:

since 2x = z

replace z with 2x

900000 = x+y+z

900000 = x+y+2x

900000 = 3x+y - eqn (1)

79750= 0.08x +0.09y+0.01z

79750 = 0.08x +0.09y+0.01(2x)

79750 = 0.08x+0.09y+0.02x

79750 = 0.10x +0.09y - eqn(2)

from eqn(1)

900000 = 3x + y

y = 900000-3x - eqn(3)

substitute eqn(3) in eqn(2)

79750 = 0.1x +0.09y

79750=0.1x + 0.09(900000-3x)

79750=0.1x+ 81000 - 0.27x

collect like terms

79750 -81000 = 0.1x-0.27x

-1250 = -0.17x

to find x divide both sides by -0.17

x = -1250/-0.17 ~= 7353

since 2x = z

2*7353 = 14706

in eqn(3)

y = 900000-3x

y= 900000-3(7353)

y = 900000-22059

y = 877941

x =7353,y= 877941,z=14706

Select all of the following that represent the part of the grid that is shaded.



A ten-by-ten grid has 7 columns shaded.



A.


70


100


B.


7


10


C.


70


10


D.


0. 07



E.


0. 7

Answers

A ten-by-ten grid has 7 columns shaded. All of the following that represent the part of the grid that is shaded are : The correct answer is (A) 70 and (B) 7.

The information given in the problem tells us that a ten-by-ten grid has 7 columns shaded. Since there are a total of 10 columns in the grid, this means that 7/10 of the columns are shaded.

To express this as a percentage, we can divide 7 by 10 and multiply by 100:

(7/10) x 100 = 70%

Therefore, 70 represents the percentage of columns that are shaded in the grid. Option (A) is correct.

Alternatively, we can express the same proportion as a decimal by dividing 7 by 10:

7/10 = 0.7

Therefore, 0.7 represents the proportion of columns that are shaded in the grid. Option (E) is incorrect because it shows 0.7 as a fraction instead of a decimal.

Option (B) is also correct because it correctly identifies the number of shaded columns as 7. Option (C) is incorrect because it includes both the percentage and the number of shaded columns, which is redundant. Option (D) is incorrect because it shows the proportion of shaded columns as a decimal with an extra zero.

To know more about grid, refer to the link below:

https://brainly.com/question/29774894#

#SPJ11

What is the product of the expression, 5x(x2)? (1 point)
25x2
10x
5x3
5x2

Answers

The product of the expression 5x(x²) is 5x³.


1. Write down the given expression: 5x(x²)
2. Apply the distributive property, which states that a(b + c) = ab + ac. In this case, we have a single term inside the parentheses, so the expression becomes: 5x * x²
3. Multiply the coefficients (numbers) together: 5 * 1 = 5
4. Multiply the variables together, which means adding the exponents since they have the same base (x): x¹* x² = x⁽¹⁺²⁾ = x³
5. Combine the result from steps 3 and 4: 5x³

The product of the expression 5x(x²) can be found by multiplying the coefficients (numbers) and adding the exponents of the variables (letters). In this case, we have 5 times x times x squared.

5 times x equals 5x, and x squared means x times x, so we can rewrite the expression as:

5x(x²) = 5x(x*x) = 5x³

So, the product of the expression 5x(x²) is 5x³.

Know more about expression here:

https://brainly.com/question/14083225

#SPJ11

Other Questions
What set of coefficients will balance the chemical equation below:___C3H8 (g) + ___O2 (g) ___CO2 (g) + ___H2O (l)A. 1,5,3,4B. 3,2,2,2C. 1,3,3,1D. 2,10,6,8 Tickets of a program at college cost $3 for general admission or $2 with student ID. If 181 people paid to see a performance and $435 was collected, how many of each type of ticket were sold. A doctor is trying to diagnose a patient with short stature. she plots the girls growth in red on the growth chart below to compare it to the typical growth rates, which are shown in black. a graph titled stature for age percentiles: girls, 2 to 20 years. the graph shows lines ranging from 60 to 70. the red line falls at 54. based on the data, which of the following is the most likely diagnosis? gigantism an underactive pituitary gland an overactive pituitary gland an overactive adrenal gland Strikes by Chinese workers against grape growers began the grape boycott.O TrueO False A path 3 feet wide surrounds a rectangular garden that has a length of 20 feet and a width of 12 feet. Findthe area of the path. Picture frames selling at $45 each have a variable cost is $15 each. The fixed costs of operating the shop are $7,500 per month. Given this data, the break-even point in number of units is: ___________ a. 650. b. 500. c. 475. d. 250. Recommendationsskill plansa mathlanguage artsga standardssixth grade ff 20 volume of cubes and rectangular prisms: word problems bbmyou have prizes to reveal! go to yafter visiting the needleton natural museum, irma was so fascinated by the reptile exhibitthat she got her very own pet lizard, irma bought a terrarium shaped like a rectangular prismfor her lizard to live in. the terrarium is 20 inches long, 15 inches wide, and 12 inches tall. which equation can you use to find the volume of the terrarium, v?20. 15 = 12vv = 20. 15. 12what is the volume of the terrarium?write your answer as a whole number or decimal. do not round. cubic inchessubmitlo my how does castiglione define the renaissance man in the courtier 6camryn will: attempt 1question 15 (3 points)a steam turbine has an efficiency of 40.0%. a steam engine has an efficiency of25.0%. suppose both devices are provided with 1000 j of thermal energy. how muchmore useful work will the steam turbine do? show your work.pa.. h. When an apple is cut and exposed to air, its flesh turns brown. this reaction is controlled by an enzyme called polyphenol oxidase. the enzyme has binding sites for oxygen and phenol, a type of organic compound in apples. the list shows the sequence of events that results in a cut apple turning brown.oxygen is introduced into damaged plant tissue.the enzyme polyphenol oxidase binds oxygen from the air and phenols from the apple.the enzyme catalyzes a reaction that changes phenols into o-quinones, a different organic molecule.o-quinones react with amino acids in the apple's flesh to produce the brown color.a student wants to investigate whether citric acid acts as an inhibitor of the enzyme polyphenol oxidase. which experiment should the student use? a. obtain a solution of o-quinone, add a small amount of phenol and citric acid solutions, and observe the color of the solution. b. obtain a solution of polyphenol oxidase, add a small amount of citric acid, and observe the color of the solution. c. place freshly cut apple slices in a dish, treat some slices with citric acid, and observe their appearance. d. place freshly cut apple slices in a dish in an oxygen-free chamber, treat some slices with citric acid, and observe their appearance. 18. Triangle GHJ has GH = 13, GJ = 15, andm/G = 74. What is the area of the triangle?SEE EXAMPLE 4 Read the sentences below. ""the flick was gr8. ull how should you rewrite these sentences for a formal movie review? ""the flick was great. youll love it."" ""the movie was okay. you might like it."" ""the movie was something that you might enjoy."" ""the film was a masterpiece that you will truly enjoy."" The mathematical phrase 5 + 2 18 is an example of a(n) 13) a 95 percent confidence interval estimate will have a margin of error that is approximately + or - 47.5 percent of the size of the population mean. true or false which fraction is the smallest? 3to4, 2/3, 10:12, 2to1 Please help these 3 are Biology questions i need a hand withFor three answers i will give 30 points no trolls Given: AB=CD, AD|| BC, BF=HD, CGE=AHF and AE=FC.Prove: BAE=DCF Biodiversidad en collombia. Xiencias sociales A major goal of the hutu-led regime in rwanda in mid-1994 was to. 14 Elodea is a plant that grows in fresh water and is often found in aquariums, Threeelodea plants were placed inside inverted test tubes filled with water. One test tubewas exposed to low light, another to medium light while another was exposed tohigh light. The table provided shows the results of this experiment. Notice that moreoxygen was produced by the elodea in high light. Based on the information provided, what is the difference in volume (in ml) ofoxygen produced between the low light and high light plants at 45 minutes?Time (min)015304560Oxygen Production (mL)Elodea in Low Elodea in Medium Elodea in HighLightLightLight0000. 40. 80. 91. 21. 71. 81. 42. 02. 51. 82. 2. 3. 2. F 1. 4G 1. 1H0. 6J 0. 4